Hi fellow students! I'm a bit confused about the question listed above. I picked A, because I thought this would reconcile the issue with sample size. But the correct answer is C, and I don't really understand why... couldn't the percentages refer to ...
First sentence I diagrammed as: wind & below 84-> pleasant. The second part of this question I diagrammed as : above 84 or no wind ->oppressive. I incorrectly got answer choice (B) it has something to do the the humidity but I was able to add ...
I don't understand why A is the correct answer. I chose C. Could someone please explain why the right answer is right and the wrong answers are wrong? Thank you
https://7sage.com/lsat_explanations/lsat-41-section-3-question-23
I understand the reasoning in the video, that just because X is preceded by Y, it doesn’t mean that Y is the necessary condition.
https://7sage.com/lsat_explanations/lsat-67-section-3-game-4/
I was not sure about the question sentence...it says "If three subzones in all are designated for retail use and a subzone in Z2 is designated for housing, then which one of the following ...
I thought the principle to be extracted from the stimulus was somewhere along the lines of, in order to gain acceptance for a theory, there needs to be ...
I cannot for the life of me figure out why C and D are wrong, can give a coherent detailed explanation for why they're wrong?
https://7sage.com/lsat_explanations/lsat-40-section-3-question-23/
Hi everyone! For this question, I understand how answer choice B wrecks the argument, making it a solid necessary assumption. It was my original answer choice, but I thought I was being tricked at the back end of the test because it seemed like a ...
S: An owner of a work of art have the ethical right to destroy that artwork if (1) they find it morally/visually distasteful or (2) caring for it becomes inconvenient. This right to ...
would it be a good way to think alternative to JY's explanation that answer choice B is wrong merely in having few as the existential indicator rather than a universal ...
Ok, so I got this MBT question wrong. I initially was going to go with AC D (the right choice) but was turned off from the second part of the answer. The whole thing reads:
"More money is spent on microwave food products that take three ...
I am confused as to why answer c is a better answer choice than a. They seem very similar, but is the answer c because there are a range of numbers opposed to using the same number (2) in answer choice a, or is there something bigger that I am missing?
I understand that (c) is a better answer choice than (b), but I couldn't rule (b) out either. Isn't the direction of public policy out of scope with regards to the stimulus? Is there something in the passage that I am missing here? Many thanks in advance!< ...
For Q4: I thought D was totally wrong and I was completely sure of it because I thought to myself, "How could we possibly know what's in the best interest for the military?" I fell for answers ...